The visitors to a certain website were questioned about their favorite soups.

If 180 people were surveyed, how many more people voted for split pea soup than for potato soup?

The Visitors To A Certain Website Were Questioned About Their Favorite Soups.If 180 People Were Surveyed,

Answers

Answer 1

Answer:

.15(180) - .10(180) = .05(180) = 9

9 more people voted for split pea soup than for potato soup.


Related Questions

to
Mrs. James, a sixth grade teacher, recorded how many minutes
each student reported reading over winter break.
Time spent reading (min.)
0
H
200.
What was the upper quartile of the time spent reading?
minutes
Submit
100
H
300.
400
+
500

Answers

The upper quartile of the time spent reading is 300.

We have,

The upper quartile, also known as the third quartile, is a measure of central tendency that divides a data set into four equal parts.

It represents the data point that separates the top 25% of the data from the bottom 75%.

Now,

From the box pot.

Median = 250

Lower quartile = 100

Upper quartile = 300

Thus,

The upper quartile is 300.

Learn more about the interquartile range here:

https://brainly.com/question/29204101

#SPJ1

1. A curve C in R3 is given by x = {3 – 1, y = -3t2 + 2, z = 8t – 2. Find parametric equations for the tangent line to C at P = (0, -1,6).

Answers

The parametric equations for the tangent line to curve C at point P are: x = -t y = 6t - 1 z = 8t + 6

To find the parametric equations for the tangent line to curve C at point P, we need to first find the derivative of the curve at P.

Taking the derivative of each component of the curve, we get:

dx/dt = -1

dy/dt = -6t

dz/dt = 8

At point P = (0, -1, 6), t = -1.

Plugging this into the derivative, we get:

dx/dt = -1

dy/dt = 6

dz/dt = 8

So, the tangent vector to curve C at point P is < -1, 6, 8 >.

To get the parametric equations for the tangent line, we can use the point-slope form: r(t) = P + t< -1, 6, 8 > Plugging in the coordinates of point P, we get: r(t) = <0, -1, 6> + t< -1, 6, 8 > Expanding this out, we get: r(t) = <-t, 6t - 1, 8t + 6>

So, the parametric equations for the tangent line to curve C at point P are: x = -t y = 6t - 1 z = 8t + 6

Learn more about parametric equations,

https://brainly.com/question/30451972

#SPJ11

The table gives a set of outcomes and their probabilities. Let A be the event "the outcome is greater than 1". Let B be the event "the outcome is greater than or equal to 2". Find P(A or B). Outcome Probability 1 0. 33 2 0. 19 3 0. 13 4 0. 31 5 0. 04 ​

Answers

The probability of event A or B occurring i.e.,  P(A or B) is 0.67.

Given the table of outcomes and their probabilities, you need to find P(A or B), where A is the event "the outcome is greater than 1" and B is the event "the outcome is greater than or equal to 2".

1: Identify the outcomes that satisfy A or B.

A: Outcomes greater than 1: {2, 3, 4, 5}

B: Outcomes greater than or equal to 2: {2, 3, 4, 5}

A or B: Outcomes greater than 1 or greater than or equal to 2: {2, 3, 4, 5}

2: Calculate the probability of each outcome in the combined set A or B.

Outcome 2: Probability 0.19

Outcome 3: Probability 0.13

Outcome 4: Probability 0.31

Outcome 5: Probability 0.04

3: Add up the probabilities of each outcome in the combined set A or B.

P(A or B) = P(2) + P(3) + P(4) + P(5)

P(A or B) = 0.19 + 0.13 + 0.31 + 0.04

P(A or B) = 0.67

Therefore, the probability of event A or B occurring, where A is "the outcome is greater than 1" and B is "the outcome is greater than or equal to 2", is 0.67.

Learn more about Probability:

https://brainly.com/question/25839839

#SPJ11

Dario, a prep cook at an Italian restaurant, spins a salad spinner and observes that it rotates with constant speed 20.0 times in 5.00 seconds and then stops spinning it. The salad spinner rotates 6.00 more times before it comes to rest. Assume that the spinner slows down with constant angular acceleration Part A Dario, a prep cook at an Italian restaurant, spins a salad spinner and observes that it rotates with constant speed 20.0 times in 5.00 seconds and then stops spinning it. The salad spinner rotates 6.00 more times before it comes to rest. Assume that the spinner slows down with constant angular acceleration. What is the magnitude of the angular acceleration of the salad spinner as it slows down? Express your answer numerically in radians per second per second. ► View Available Hint(s) a = 8.38 radians/s2 Submit Previous Answers Correct Part B How long does it take for the salad spinner to come to rest? Express your answer numerically in seconds. View Available Hint(s) EVO AEO ? t = S Submit

Answers

Part a. The magnitude of the angular acceleration of the salad spinner as it slows down is 4.00 radians/s².

Part b. It takes 1.00 seconds for the salad spinner to come to rest.

Part A:

The initial angular velocity of the spinner is given by:

ω1 = 20.0 rotations / 5.00 s = 4.00 rotations/s

The final angular velocity of the spinner is zero.

The number of rotations between the initial and final angular velocities is:

Δθ = 6.00 rotations

Using the equation of motion for rotational kinematics with constant angular acceleration:

Δθ = 1/2 α t^2 + ω1 t

where α is the angular acceleration, and t is the time it takes to stop spinning.

At the final angular velocity, ω2 = 0, so we can rearrange the equation to solve for t:

t = ω1 / α

Substituting the given values:

Δθ = 6.00 rotations

ω1 = 4.00 rotations/s

t = (4.00 rotations/s) / α

Solving for α:

Δθ = 1/2 α t^2 + ω1 t

6.00 rotations = 1/2 α (t^2) + (4.00 rotations/s) t

Substituting t = (4.00 rotations/s) / α:

6.00 rotations = 1/2 α [(4.00 rotations/s) / α]^2 + (4.00 rotations/s) [(4.00 rotations/s) / α]

6.00 rotations = 8.00 rotations + 16.00 rotations/s^2 / α

α = 16.00 rotations/s^2 / (6.00 rotations - 8.00 rotations)

α = 8.00 rotations/s^2 / 2.00 rotations

α = 4.00 radians/s^2

Therefore, the magnitude of the angular acceleration of the salad spinner as it slows down is 4.00 radians/s^2.

Part B:

Using the equation of motion for rotational kinematics with constant angular acceleration:

ω2 = ω1 + α t

At the final angular velocity, ω2 = 0, so we can rearrange the equation to solve for t:

t = -ω1 / α

Substituting the given values:

ω1 = 4.00 rotations/s

α = 4.00 radians/s^2

t = -(4.00 rotations/s) / (4.00 radians/s^2)

t = -1.00 s

Since the time cannot be negative, we take the absolute value of t:

t = 1.00 s

Therefore, it takes 1.00 seconds for the salad spinner to come to rest.

To know more about angular acceleration visit;

brainly.com/question/29428475

#SPJ11

Question 10(Multiple Choice Worth 2 points)
(Comparing Data MC)

The box plots display measures from data collected when 20 people were asked about their wait time at a drive-thru restaurant window.

A horizontal line starting at 0, with tick marks every one-half unit up to 32. The line is labeled Wait Time In Minutes. The box extends from 8.5 to 15.5 on the number line. A line in the box is at 12. The lines outside the box end at 3 and 27. The graph is titled Super Fast Food.

A horizontal line starting at 0, with tick marks every one-half unit up to 32. The line is labeled Wait Time In Minutes. The box extends from 9.5 to 24 on the number line. A line in the box is at 15.5. The lines outside the box end at 2 and 30. The graph is titled Burger Quick.

Which drive-thru typically has more wait time, and why?

Burger Quick, because it has a larger median
Burger Quick, because it has a larger mean
Super Fast Food, because it has a larger median
Super Fast Food, because it has a larger mean

Question 11(Multiple Choice Worth 2 points)
(Creating Graphical Representations MC)

The number of carbohydrates from 10 different tortilla sandwich wraps sold in a grocery store was collected.

Which graphical representation would be most appropriate for the data, and why?

Circle chart, because the data is categorical
Line plot, because there is a large set of data
Histogram, because you can see each individual data point
Stem-and-leaf plot, because you can see each individual data point
Question 12(Multiple Choice Worth 2 points)

Answers

The answer to the first question is: Burger Quick, because it has a larger median.

The answer to the second question is: Histogram, because it can show the frequency distribution of the continuous variable "number of carbohydrates" and group the data into intervals.

What is a histogram?

A histogram is a graphical representation of a distribution of numerical data. It consists of a series of bars, where each bar represents a range of values of the data and the height of the bar indicates the frequency or count of data points falling within that range.

In a histogram, the horizontal axis represents the range of values or intervals of the data, called bins, and the vertical axis represents the frequency or count of data points falling within each bin. The bins can be of equal or unequal width depending on the nature of the data.

Learn more about histogram on:

https://brainly.com/question/2962546

#SPJ1

Look at picture please

Answers

Answer:

135.6 cubic centimeters

Step-by-step explanation:

To find the volume of soda in the cylindrical glass, we need to first find the volume of the glass and then multiply it by 60% to get the volume of soda. The formula for the volume of a cylinder is V = πr^2h, where r is the radius and h is the height.

Substituting the given values, we get:

V = π(3 cm)^2(8 cm) = 72π cm^3

To find the volume of soda, we multiply the volume of the glass by 60% or 0.6:

Volume of soda = 0.6 x 72π cm^3 = 43.2π cm^3

Rounding to the nearest tenth, we get:

Volume of soda ≈ 135.6 cm^3

Therefore, there are approximately 135.7 cubic centimeters of soda in the glass

Answer: 226.3

Step-by-step explanation:

Hello! Here is how to solve this problem.

The formula for volume of a cylinder is V(c) = (B)(h)

The height is 8 cm, and the radius is 3 cm. The base is a circle, so the base area will be 22/7(r)^2. Pi is represented as 3.14 or 22/7, but 22/7 is more accurate.

V(c) = 22/7((3)^2)(8)
V(c) = 22/7(9 x 8)
V(c) = 22/7(72)

V(c) = 226.285714.

So, rounded to the nearest tenth, the volume of the soda can is 226.3 cm^3.

Tips for you:

1. Round to the nearest tenth
2. Use 22/7 for pi unless it says to use 3.14 for pi or use the symbol for pi instead of solving further.

Hope this helps, have a great day!

Brian has two cubes.

. The first cube has a volume of 125 cm3.

. The second cube has a volume of 343 cm3.


What is the difference in the area of one face of the second cube and the area of one face of the first cube?


A. 2 cm2

B. 24 cm2

C. 49 cm2

D. 218 cm2
Help asap

Answers

If the first cube has a volume of 125 cm³ and the second cube has a volume of 343 cm³, the difference in the area of one face of the second cube and the area of one face of the first cube is 24cm². The answer is B. 24 cm².

To find the difference in the area of one face of each cube, we first need to find the side length of each cube. Since the volume of a cube is equal to the side length cubed (V = s³), we can find the side length by taking the cube root of the volume.

For the first cube:
Volume = 125 cm³
Side length = cube root of 125 = 5 cm

For the second cube:
Volume = 343 cm³
Side length = cube root of 343 = 7 cm

Next, we find the area of one face of each cube. The area of one face of a cube is equal to the side length squared (A = s²).

Area of one face of the first cube:
A1 = 5² = 25 cm²

Area of one face of the second cube:
A2 = 7² = 49 cm²

Finally, find the difference in the area of one face of each cube:
Difference = A2 - A1 = 49 cm² - 25 cm² = 24 cm²

The answer is B. 24 cm².

More on cube area: https://brainly.com/question/14994710

#SPJ11

Dominic and his parents plan to share the cost of his college education. The annual



tuition cost for the college he wants to attend is $7,260 per year. His parents will pay 80%



of the annual tuition. He has one year to save his portion of the first year's tuition. What is



the minimum monthly amount he needs to save?

Answers

Dominic will need to save a minimum of $121 per month to pay for his portion of the first year's tuition.

Dominic's parents will pay 80% of the annual tuition cost, which is:

0.8 x $7,260 = $5,808

So, Dominic will need to pay the remaining 20% of the tuition cost, which is:

0.2 x $7,260 = $1,452

Since Dominic has one year to save his portion of the tuition, he will need to save:

$1,452 / 12 months = $121 per month

Therefore, Dominic will need to save a minimum of $121 per month to pay for his portion of the first year's tuition.

To know more about cost , refer here:

https://brainly.com/question/30045916#

#SPJ11

Find the product of d = ba. d11 = d12 = d21 = d22 =

Answers

Given d = ba, we can write:

d11 = b1a1 + b2a3

d12 = b1a2 + b2a4

d21 = b3a1 + b4a3

d22 = b3a2 + b4a4

To find the product of d, we need to find the values of b and a such that d11 = d12 = d21 = d22.

Let's assume that d11 = d12 = d21 = d22 = x. Then, we have:

b1a1 + b2a3 = x

b1a2 + b2a4 = x

b3a1 + b4a3 = x

b3a2 + b4a4 = x

We can solve for b1, b2, b3, and b4 in terms of a1, a2, a3, and a4:

b1 = (x - b2a3)/a1

b2 = (x - b1a1)/a3

b3 = (x - b4a3)/a1

b4 = (x - b3a1)/a3

Substituting these values of b1, b2, b3, and b4 into the equation d = ba, we get:

d11 = x = a1(x - b1a3)/a1 + a3(x - b2a1)/a3

= x - b1a3 + x - b2a1

= 2x - (b1a3 + b2a1)

d12 = x = a2(x - b1a3)/a1 + a4(x - b2a1)/a3

= (a2/a1)x - (b1a2 + b2a4) + (a4/a3)x - (b1a4 + b2a4)

= x - (b1a2 + b2a4)

d21 = x = a1(x - b3a3)/a1 + a3(x - b4a1)/a3

= (a1/a1)x - (b3a3 + b4a3) + (a3/a3)x - (b3a1 + b4a3)

= x - (b3a1 + b4a3)

d22 = x = a2(x - b3a3)/a1 + a4(x - b4a1)/a3

= (a2/a1)x - (b3a2 + b4a4) + (a4/a3)x - (b3a4 + b4a4)

= x - (b3a2 + b4a4)

We can rewrite these equations in matrix form as:

| 2 -a3-a1 0 0 || x | | b1a3 + b2a1 |

| 0 a2 0 -a4 || | = | b1a2 + b2a4 |

| -a3-a1 0 2 -a1 || | | b3a1 + b4a3 |

| 0 -a4 -a1 2 || | | b3a2 + b4a4 |

To solve for x, we need to invert the matrix on the left and multiply it by the vector on the right:

| x | | 2 -a3-a1 0

To know more about multiply refer here

httpsbrainly.42663642#

#SPJ11

Chris works at a book story she earn $7. 50 per h hour plus a $2 bonus for each book she sells chris sood 15 books she want to earn the minimum of $300 which Inequality represents the situation in what quantities are true for h

Answers

Chris must work at least 36 hours to earn a minimum of $300, assuming she sells 15 books and earns the $2 bonus for each book sold.

The inequality that represents the situation is: 7.50h + 2(15) ≥ 300 where "h" represents the number of hours Chris works, and "2(15)" represents the bonus earned for selling 15 books.

The left-hand side of the inequality calculates Chris's total earnings, which is the product of her hourly wage of $7.50 and the number of hours worked, plus the bonus earned for selling 15 books.

The inequality states that the total earnings must be greater than or equal to $300, which is the minimum amount Chris wants to earn. To solve the inequality, we can simplify it by first multiplying 2 and 15 to get 30: 7.50h + 30 ≥ 300

We can isolate "h" by subtracting 30 from both sides: 7.50h ≥ 270. we can solve for "h" by dividing both sides by 7.50: h ≥ 36.

Learn more about inequality here:

https://brainly.com/question/28823603

#SPJ4

Which amount should be entered for the balance after the atm withdrawal on 6/2/18?a) $1346.42b) $1456.42c) $1536.42d) $1756.42

Answers

Unfortunately, there is no way to answer this question without more information.

We would need to know the amount of the ATM withdrawal, as well as the starting balance before the withdrawal, in order to calculate the ending balance.

For example, if the starting balance was $1500 and the withdrawal was for $50, the ending balance would be $1450.

However, if the starting balance was $1300 and the withdrawal was for $50, the ending balance would be $1250.

Without this information, we cannot determine the correct answer.

To know more about balance refer here

https://brainly.com/question/37441951#

#SPJ11

Write the number in standard form. (8 × 10) + (1 × 1/10 ) + (6 × 1/1000 )

Answers

We can simplify the given expression first and then write it in standard form.

8 × 10 = 80

1 × 1/10 = 1/10

6 × 1/1000 = 6/1000 = 3/500

Adding these three values, we get:

80 + 1/10 + 3/500

To write this in standard form, we need to express it as a single number multiplied by a power of 10. We can do this by finding a common denominator for the fractions and adding them:

80 + 50/500 + 3/500 = 80 + 53/500

Now, we can write this as:

80.106

To express this in standard form, we move the decimal point to the left until there is only one non-zero digit to the left of the decimal point. We moved the decimal point 3 places to the left to get:

8.0106 × 10^1

Therefore, the number in standard form is 8.0106 × 10^1.

WILL MARK BRAINLIEST 100 POINTS

Answers

Answer:

The answer is the fourth option - 0.63

Step-by-step explanation:

find the area of each polygon below b=6 h=9 ft h =10cm b = 8 h=8m b=9m

Answers

To find the area of a polygon, we need to know the length of its base and its height. Once we have these measurements, we can use the formula:

Area = (1/2) x Base x Height

where "Base" refers to the length of the base of the polygon, and "Height" refers to the length of a perpendicular line drawn from the base to the opposite vertex.

Using this formula, we can find the areas of the polygons with the given measurements:

For the first polygon with base b=6 ft and height h=9 ft, we have:

Area = (1/2) x 6 ft x 9 ft = 27 ft^2

For the second polygon with base b=10 cm and height h=8 m, we need to convert the height to centimeters to keep the units consistent:

Height = 8 m x 100 cm/m = 800 cm

Then, we can calculate the area as:

Area = (1/2) x 10 cm x 800 cm = 4000 cm^2

For the third polygon with base b=8 m and height h=8 m, we have:

Area = (1/2) x 8 m x 8 m = 32 m^2

For the fourth polygon with base b=9 m and height h=5 m, we have:

Area = (1/2) x 9 m x 5 m = 22.5 m^2

Therefore, the areas of the polygons are:

- 27 ft^2
- 4000 cm^2
- 32 m^2
- 22.5 m^2

dont guess, due in a few minuets

Answers

For a proper use of unit multipliers to convert 24 square feet per minute, the right choice is A.

How to determine conversion?

The proper use of unit multipliers to convert 24 square feet per minute to square inches per second is:

24 ft²/1 min × 12 in/1 ft × 12 in/1 ft × 1 min/60 sec = (24 × 12 × 12)/(1 × 1 × 60) in²/sec

Thus, when these conversion factors are multiplied by the specified value of  24 ft²/1 min:

24 ft²/1 min . 12 in/1 ft . 12 in/1 ft . 1 min/60 sec

= (24 x 12 x 12) in² / (1 x 1 x 1) min x (1 x 1 x 60) sec

= 4,608 in²/sec

Therefore, the correct answer choice is:

24 ft²/1 min . 12 in/1 ft . 12 in/1 ft . 1 min/60 sec.

Find out more on unit multipliers here: https://brainly.com/question/13736702

#SPJ1

this 3 questions please????????
Consider the following function
f(x) = x^2/x^2 -9
Find the criticat number of renter your antwers as a comma-separated list) Find the open intervals on which the function is increasing or decreasing. (Enter your answers using interval notation. If an answer does not exist, enter DNE.)

Answers

Critical numbers: 0, -3, 3
Increasing intervals: (-∞, -3), (3, ∞)
Decreasing intervals: (-3, 0), (0, 3)

Let's analyze the given function and find the critical numbers, as well as the intervals where it is increasing or decreasing.

Function: f(x) = x^2 / (x^2 - 9)

1. Find the critical numbers:
To find the critical numbers, we need to compute the first derivative of the function and set it equal to zero or identify where it's undefined.

f'(x) = (2x(x^2 - 9) - x^2(2x)) / (x^2 - 9)^2 = (2x^3 - 18x - 2x^3) / (x^2 - 9)^2 = -18x / (x^2 - 9)^2

Setting f'(x) equal to zero:
-18x = 0
x = 0

Now, let's check where f'(x) is undefined:
x^2 - 9 = 0
x = ±3

Thus, the critical numbers are 0, -3, and 3.

2. Find the open intervals where the function is increasing or decreasing:
To determine the intervals, we will examine the sign of f'(x) in the intervals determined by the critical numbers: (-∞, -3), (-3, 0), (0, 3), (3, ∞).

Interval (-∞, -3): Choose x = -4, then f'(-4) > 0. So, f(x) is increasing on this interval.

Interval (-3, 0): Choose x = -1, then f'(-1) < 0. So, f(x) is decreasing on this interval.

Interval (0, 3): Choose x = 1, then f'(1) < 0. So, f(x) is decreasing on this interval.

Interval (3, ∞): Choose x = 4, then f'(4) > 0. So, f(x) is increasing on this interval.

Your answer:
Critical numbers: 0, -3, 3
Increasing intervals: (-∞, -3), (3, ∞)
Decreasing intervals: (-3, 0), (0, 3)

To learn more about critical numbers, refer below:

https://brainly.com/question/29743892

#SPJ11

The school physics class has built a trebuchet (catapult) that is big enough to launch a watermelon. the math class has created the function h(t) = -16( t - 5)2 + 455 to model the height, in feet, after t seconds, of a watermelon launched into the air from a hilltop near the school the x - intercepts of this function are (-0.33 , 0) and (10.33 , 0)

the watermelon is hitting the ground at around ____ seconds

Answers

The watermelon is hitting the ground at around 10.33 seconds.

To find out when the watermelon hits the ground, we need to look for the time when the height of the watermelon is zero. This is because the watermelon will be on the ground at that point.

The x-intercepts of the function h(t) give us the times when the height is zero. So, we know that the watermelon will hit the ground at t = -0.33 seconds and t = 10.33 seconds.

However, the negative value doesn't make sense in this context, so we can ignore that solution. Therefore, the watermelon is hitting the ground at around 10.33 seconds.

Learn more about Speed and Time:  https://brainly.com/question/31756299

#SPJ11

Joe’s fish store has 18 goldfish. The fish are on 3 aquariums. The same number of goldfish are in each aquarium. How many goldfish are in each aquarium?

Answers

Answer:

There are 6 goldfish in each of the 3 aquariums at Joe's fish store.

Step-by-step explanation:

To find out how many goldfish are in each aquarium, we can divide the total number of goldfish by the number of aquariums:

18 goldfish ÷ 3 aquariums = 6 goldfish per aquarium

Therefore, there are 6 goldfish in each of the 3 aquariums at Joe's fish store.

Now that you learned how to calculate the probabilities of each player winning the "maximum game" in the video, let's look at the probabilities of another game. this is how it works: we roll two dice and calculate the multiplication of the two numbers we rolled. --if it is a multiple of 6, i win --if it is not a multiple of 6, you win. here is an example: if you get 3 and 4, the multiplication is 12. twelve is a multiple of 6, so i win! 1. which player would win if you get 2 and 5 in the dice? me or you? 2. which player would win if you get 4 and 2 in the dice? 3. which player would win if you get 1 and 6 in the dice?

Answers

1) If you get 2 and 5, the multiplication is 10, which is not a multiple of 6. so, you would win.

2) If you get 4 and 2, the multiplication is 8, which is not a multiple of 6. so, you would win.

3) If you get 1 and 6, the multiplication is 6, which is a multiple of 6. so, I would win.

1) How to find  the probability?

The question asks about probability  which player would win if the numbers rolled are 2 and 5. To answer this, we calculate the product of 2 and 5, which is 10. Since 10 is not a multiple of 6, the person who did not roll the dice (i.e., "you") would win.

2) How to find  the probability?

The question asks which player would win if the numbers rolled are 4 and 2. We calculate the product of 4 and 2, which is 8. Since 8 is not a multiple of 6, "you" would win again.

3) How to find  the probability?

The question asks which player would win if the numbers rolled are 1 and 6. We calculate the product of 1 and 6, which is 6. Since 6 is a multiple of 6, the person who rolled the dice (i.e., "me") would win.The game described in the question involves rolling two dice and calculating the multiplication of the two numbers rolled. The outcome of the game depends on whether the product is a multiple of 6 or not.

The solution also provides a general explanation of how to calculate the probability of rolling a multiple of 6 with two dice. To do this, we count the number of ways to roll each multiple of 6 (there are two ways to roll a 6, one way to roll a 12, and no ways to roll an 18) and divide by the total number of possible outcomes (which is 36, since there are 6 possible outcomes for each die and 6*6=36 possible combinations of two dice). This gives us a probability of 1/12, or approximately 0.0833, for rolling a multiple of 6. We can then calculate the probability of not rolling a multiple of 6 by subtracting this probability from 1, which gives us 11/12, or approximately 0.9167.

Learn more about probability

brainly.com/question/11234923

#SPJ11

C(x) - 1700 + 5x + 0.08x² + 0.0004x³. (a) Find the marginal cost function (b) Find C'(100) C'(100) What does this predict? The exact cost of the 101st pair of jeans The exact cost of the 100th pair of jeans The approximate cost of the 100th pair of jeans The approximate cost of the 101st pair of jeans The exact cast of the 99th pair of jeans. (c) Find the difference between C'(100) and the actual cost of manufacturing the 101st pair of jeans (Round your answer to two decimal places)

Answers

(a) The marginal cost function is C'(x) = 5 + 0.16x + 0.0012x².

(b) C'(100) = 21.00. This predicts the approximate cost of the 101st pair of jeans.

(c) The difference between C'(100) and the actual cost of manufacturing the 101st pair of jeans is $20.51.

(a) To find the marginal cost function, we need to take the derivative of the cost function C(x) with respect to x:

C'(x) = 5 + 0.16x + 0.0012x²

(b) To find C'(100), we substitute x = 100 into the marginal cost function:

C'(100) = 5 + 0.16(100) + 0.0012(100)²
C'(100) = 21.00

This predicts the approximate cost of the 101st pair of jeans, as the marginal cost function tells us the cost of producing one additional unit (pair of jeans) at a given quantity. So, we can use C'(100) to estimate the cost of producing the 101st pair of jeans.

To find the approximate cost of the 100th pair of jeans, we can substitute x = 100 into the cost function C(x):

C(100) = -1700 + 5(100) + 0.08(100)² + 0.0004(100)³
C(100) = $2330

So, the approximate cost of the 100th pair of jeans is $2330.

To find the approximate cost of the 101st pair of jeans, we can add C'(100) to the cost of producing 100 pairs of jeans:

C(100) + C'(100) = $2330 + $21.00
C(101) ≈ $2351

So, the approximate cost of the 101st pair of jeans is $2351.

To find the exact cost of the 99th pair of jeans, we can substitute x = 99 into the cost function C(x):

C(99) = -1700 + 5(99) + 0.08(99)² + 0.0004(99)³
C(99) = $2288.44

So, the exact cost of the 99th pair of jeans is $2288.44.

(c) To find the difference between C'(100) and the actual cost of manufacturing the 101st pair of jeans, we need to subtract the cost of producing 100 pairs of jeans from the cost of producing 101 pairs of jeans:

C(101) - C(100) = [-1700 + 5(101) + 0.08(101)² + 0.0004(101)³] - [-1700 + 5(100) + 0.08(100)² + 0.0004(100)³]
C(101) - C(100) = $20.51

So, the difference between C'(100) and the actual cost of manufacturing the 101st pair of jeans is $20.51.

For more such questions on Marginal cost function.

https://brainly.com/question/29583181#

#SPJ11

the cost of a taxi ride is $3.00 plus $0.75 for every 0.5km. represent the relation in a table (up to 10km), a graph and an equation

Answers

The equation for the cost of a taxi ride as c = 0.75(d - 0.5) + 3.

What is distance travelled?

Distance traveled refers to the total distance covered by an object or person over a certain period of time or within a given context. It can be measured in units such as meters, kilometers, miles, or any other unit of length.

According to question:

Equation:

Let d be the distance travelled in kilometres, and let c be the cost in dollars. Then we can write the equation for the cost of a taxi ride as:

c = 0.75(d - 0.5) + 3

This equation takes into account the initial $3.00 fee, as well as the additional $0.75 for every 0.5km

Table:

Distance (km) Cost ($)

0.5 3.38

1 3.75

1.5 4.13

2 4.50

2.5 4.88

3 5.25

3.5 5.63

4 6.00

4.5 6.38

5 6.75

5.5 7.13

6 7.50

6.5 7.88

7 8.25

7.5 8.63

8 9.00

8.5 9.38

9 9.75

9.5 10.13

10 10.50

Graph:

The x-axis represents the distance in kilometers, and the y-axis represents the cost in dollars. We start the graph at (0, 3), and then plot a point at (0.5, 3.75), (1, 4.5), (1.5, 5.25), and so on, until we get to (10, 18). We then draw a line connecting all of these points to create a line graph.

To know more about distance travelled visit:

https://brainly.com/question/29055485

#SPJ1

Ignacio chooses a plant at random that does not have a white bloom. What is the probability of the complement of the event? Express your answer as a fraction in simplest form

Answers

The probability of the complement of the event of Ignacio chooses a plant at random that does not have a white bloom is 0.7692.

The probability of an occurrence is a figure that represents how likely it is that the event will take place. In terms of percentage notation, it is stated as a number between 0 and 1, or between 0% and 100%. The higher the likelihood, the more probable it is that the event will take place.

Probability is a way to gauge how likely something is to happen. Several things are difficult to forecast with absolute confidence. With it, we can only make predictions about the likelihood of an event happening, or how likely it is.

The probability that Ignacio chooses the plant which have white bloom in it is,

P = number of white bloom / total number of flowers

P = 21 / 91

So the probability that the chosen flower is not white is,

1 - P = 1 - 21/91 = 70/91 = 0.7692.

Therefore, the probability of not choosing white is 0.7692.

Learn more about Probability:

https://brainly.com/question/30859510

#SPJ4

The amount of money required to support a band field trip is directly proportional to the number of members attending the field trip and inversely


proportional with the fundraising money each member raised. If 100 members attend the field trip and each member raised $15. 00 through fundraising, the


field trip would cost $2,000. How much would the field trip cost if 150 members attend and each member raises the same amount through fundraising?

Answers

Cost of field trip remains $2,000 with 150 members

Field trip cost with 150 members?

We can set up a proportion to solve for the cost trip with 150 members attending:

Let x be the cost of the field trip for 150 members attending.

The amount of money required is directly proportional to the number of members attending, so we can write:

[tex]100 : 150 = 2000 : x[/tex]

The amount of money required is also inversely proportional to the fundraising money each member raised. Each member raised $15.00 through fundraising, so we can write:

[tex]15 : 15 = x : 2000[/tex]

Simplifying the second proportion, we have:

[tex]1 : 1 = x/2000[/tex]

Multiplying both sides by 2000, we get:

[tex]x = 2000[/tex]

Therefore, the cost of the field trip with 150 members attending and each member raising $15.00 through fundraising would also be $2,000.

Learn more about  cost trip

brainly.com/question/18249048

#SPJ11

5 points


You need to change a blown outdoor lightbulb on your house. The bulb is 5m up, but you have a 1m reach when you are on the top rung of the ladder. If you need 3m of


space off the house for the ladder's base for stability, what is the minimum height of the ladder in meters?

Answers

The minimum height of the ladder needed to change the blown outdoor lightbulb is 5 meters.

To determine the minimum height of the ladder needed to change a blown outdoor lightbulb that is 5m up, we need to consider the following terms:

1. The bulb's height (5m)


2. Your reach when on the top rung of the ladder (1m)


3. The required space off the house for the ladder's base for stability (3m)

First, subtract your reach from the bulb's height: 5m - 1m = 4m. This means the ladder needs to reach at least 4 meters up the wall.

Next, we need to use the Pythagorean theorem to find the ladder's minimum height. The theorem states that in a right-angled triangle, the square of the length of the hypotenuse (the ladder) is equal to the sum of the squares of the lengths of the other two sides (the distance from the house and the height up the wall).

Let's denote the ladder's height as L, the distance from the house as A (3m), and the height up the wall as B (4m).

According to the Pythagorean theorem, we have:
L² = A² + B²

Substitute the values for A and B:
L² = (3m)² + (4m)²
L² = 9m² + 16m²
L² = 25m²

Now, find the square root to get the minimum height of the ladder:
L = √25m²
L = 5m

So, the minimum height of the ladder needed to change the blown outdoor lightbulb is 5 meters.

To know more about minimum height refer here:

https://brainly.com/question/15240079

#SPJ11

Mr. Jones has $410,000 in a retirement account that earns 3. 85% simple interest each year. Find the amount of interest earned by this investment if it is in there for 5 years

Answers

The amount of interest earned by Mr. Jones's retirement account over 5 years is $79,025.

Mr. Jones has invested $410,000 in a retirement account that earns 3.85% simple interest per year. Simple interest is calculated by multiplying the principal amount by the annual interest rate and the time period in years.

In this case, the time period is 5 years. Using the formula for simple interest, we can calculate the amount of interest earned on this investment as:

I = P * r * t = $410,000 * 0.0385 * 5 = $79,025

Therefore, the amount of interest earned by Mr. Jones's retirement account over 5 years is $79,025. This means that the total value of his retirement account after 5 years would be $489,025 ($410,000 + $79,025).

Learn more about interest:

brainly.com/question/30393144

#SPJ4

Help with question in photo please

Answers

Answer:

  124°

Step-by-step explanation:

You want the measure of the angle marked (4+10x) where chords cross. The chords intercept arcs marked (9x+20) and (10x).

Angle relation

The measure of the angle where chords cross is the average of the measures of the intercepted arcs.

  ((9x +20) +(10x))/2 = 4 +10x

  19x +20 = 20x +8 . . . . . . . . . multiply by 2

  12 = x . . . . . . . . . . . . . . subtract (19x+8)

The angle at E is ...

  4 +10(12) = 124

The measure of angle DEC is 124°.

__

Additional comment

Arc DC is 128°; arc BU is 120°.

On a coordinate plane, a line segment has endpoints P(6,2) and Q(3. 8). 9. Point M lies on PQ and divides the segment so that the ratio of PM-MQ is 2-3. What are the coordinates of point M?​

Answers

The coordinates of point M are:

Coordinates of M = (4.5 + sqrt(34)/5, 5)
Coordinates of M = (5.86, 5)

To find the coordinates of point M, we first need to find the coordinates of the midpoint of segment PQ.

The midpoint of a line segment with endpoints (x1, y1) and (x2, y2) is ((x1 + x2)/2, (y1 + y2)/2).

Using this formula, we can find the midpoint of PQ as follows:

Midpoint = ((6 + 3)/2, (2 + 8)/2)
Midpoint = (4.5, 5)

Now we can use the fact that PM/MQ = 2/3 to find the coordinates of point M.

Let's start by finding the distance between P and the midpoint:

Distance between P and midpoint = sqrt((4.5 - 6)^2 + (5 - 2)^2)
Distance between P and midpoint = sqrt(4.25)

Since PM/MQ = 2/3, we know that PM is 2/5 of the total distance between P and Q, and MQ is 3/5 of the total distance.

Let's call the total distance between P and Q "d". Then we have:

PM = (2/5)d
MQ = (3/5)d

We can use these expressions to find the distance between M and the midpoint:

Distance between M and midpoint = PM - MQ
Distance between M and midpoint = (2/5)d - (3/5)d
Distance between M and midpoint = -(1/5)d

Finally, we can use the distance formula to find the coordinates of point M:

Coordinates of M = (4.5, 5) + (Distance between M and midpoint in the x direction, Distance between M and midpoint in the y direction)

In other words, the x-coordinate of M is 4.5 plus the distance between M and the midpoint in the x direction, and the y-coordinate of M is 5 plus the distance between M and the midpoint in the y direction.

We already know that the distance between M and the midpoint in the y direction is 0 (since M lies on the same horizontal line as the midpoint), so we only need to find the distance between M and the midpoint in the x direction:

Distance between M and midpoint in the x direction = sqrt((1/5)d^2)

Since d is just the distance between P and Q, we can find it using the distance formula:

d = sqrt((6 - 3)^2 + (2 - 8)^2)
d = sqrt(34)

So we have:

Distance between M and midpoint in the x direction = sqrt((1/5)(sqrt(34))^2)
Distance between M and midpoint in the x direction = sqrt(34)/5

To know more about coordinates refer to

https://brainly.com/question/17206319

#SPJ11

PLEASE HELP DUE TODAY



2. The data in the table represent the training times (in seconds) for Adam and Miguel.



Adam 103 105 104 106 100 98 92 91 97 101


Miguel 88 86 89 93 105 85 92 96 97 94



(a) All of the training times of which person had the greatest spread? Explain how you know.


(b) The middle 50% of the training times of which person had the least spread? Explain how you know.


(c) What do the answers to Parts 2(a) and 2(b) tell you about Adam’s and Miguel’s training times?

Answers

(a) Miguel had the greatest spread in training times.

(b) Adam had the least spread in the middle 50% of training times.

(c) Miguel's training times had a greater range, indicating more variability, while Adam's training times were more consistent and tightly grouped.

(a) Who had the greatest spread?(b) Who had the least spread?(c) how do the answers indicate?

(a) To determine which person had the greatest spread, we need to compare the range or variability of their training times. By observing the given data, we can see that Adam's training times range from 92 to 106, resulting in a spread of 14. On the other hand, Miguel's training times range from 85 to 105, resulting in a spread of 20. Therefore, Miguel had the greatest spread of training times.

(b) To determine which person had the least spread in the middle 50% of training times, we need to compare the interquartile range (IQR). By calculating the IQR, we find that Adam's IQR is 9 (from the 25th to the 75th percentile), whereas Miguel's IQR is 7. Since Adam's IQR is greater, it means Miguel had the least spread in the middle 50% of training times.

(c) The answers to parts (a) and (b) indicate that while Miguel had a greater spread of training times overall, Adam's training times had a greater spread in the middle 50%. This suggests that Adam's training times were more concentrated around the median, while Miguel's training times were more spread out.

Learn more about variability

brainly.com/question/17344045

#SPJ11

a) All of the training times of Adam had the greatest spread.

(b) The middle 50% of the training times of Adam had the least spread.

(c) The answers to Parts (a) and (b) tell us that Adam's training performance may be more stable within that middle 50%, while Miguel's performance is more variable.

(a) Adam's training times had the greatest spread.

To determine this, we can calculate the range of the data sets. For Adam, the range is 106-92 = 14 seconds, while for Miguel, the range is 105-85 = 20 seconds. However, a better measure of spread is the interquartile range (IQR), which focuses on the middle 50% of the data. For Adam, the IQR is 101-97 = 4 seconds, while for Miguel, the IQR is 96-89 = 7 seconds. In both cases, Miguel's data has a greater spread.

(b) Adam's training times had the least spread for the middle 50% of the data. This is demonstrated by the IQR, as mentioned above. For Adam, the IQR is 4 seconds, while for Miguel, it is 7 seconds.

(c) The answers to Parts 2(a) and 2(b) tell us that while Miguel's overall training times have a greater spread, the middle 50% of Adam's training times are more consistent, with less variation. This suggests that Adam's training performance may be more stable within that middle 50%, while Miguel's performance is more variable.

Learn more about Spread:

https://brainly.com/question/12750165

#SPJ11

Clara's class is preparing for a field trip. Her teacher purchased bottled water for the trip and asked Clara to stock a cooler with 2 bottles for every student who is going. 5 of the students didn't turn in permission slips and aren't going on the trip. So, Clara stocks the cooler with 38 bottles of water.

Which equation can you use to find the total number of students, n, in Clara's class?

Answers

The equation that can be used to find the total number of students would be n = 19 + 5.

How to find the equation ?

It is acknowledged that Clara provided 2 bottles per each pupil joining her on the excursion. Denote, by using ‘x’, the quantity of learners present; we can then inscribe the succeeding formula:

2x = 38

By resolving this mathematical principal, the total number of students attending the event is revealed.

x = 38 / 2

x = 19

Currently, 19 individuals are confirmed to embark upon the outing, due to five individuals failing to furnish required consent forms and will be absent. Subsequently, we may deduce the quantity of pupils (designated as ‘n’) in Clara’s class:

n = 19  (students going on the trip ) + 5 ( students not going )

n = 19 + 5

Find out more on equations at https://brainly.com/question/25976025

#SPJ1

1. In a nation centre, the administrative fee is RM30 per student. The tuition fee is RM45 per subject for language subjects and RM40 per subject for other subjects.

(a) Express the total payment. J. for a student who registers for m language subjects and n other subjects.

(b) Zaleha registers for 3 language subjects and 2 other subjects. How much does she have to pay?

(c) Chan pays RM280 when she registers for 2 language subjects and p other subjects. Find the value of p.
2. The diagram shows a right pyramid with a square base.

(a) Form a formula by using the surface area of the pyramid, L,as the subject of the formula.

(b) Calculate the surface area of the pyramid if a 10 and b = 12.

(c) If L=192 and a=b, find the value of a​

Answers

Answer:

Step-by-step explanation:

(a) The total payment for a student who registers for m language subjects and n other subjects can be expressed as:

Total payment = Administrative fee + Tuition fee for language subjects + Tuition fee for other subjects

Total payment = RM30 + RM45m + RM40n

Total payment = RM30 + 45m + 40n

(b) For Zaleha who registers for 3 language subjects and 2 other subjects, the total payment can be calculated as:

Total payment = RM30 + (3 x RM45) + (2 x RM40)

Total payment = RM30 + RM135 + RM80

Total payment = RM245

(c) Chan pays RM280 when she registers for 2 language subjects and p other subjects. We can use the formula derived in part (a) to find the value of p:

Total payment = RM30 + (2 x RM45) + (p x RM40)

RM280 = RM30 + RM90 + RM40p

RM280 - RM120 = RM40p

RM160 = RM40p

p = 4

Therefore, Chan registers for 2 language subjects and 4 other subjects.

(a) The surface area of a right pyramid with a square base can be calculated as:

L = base area + 1/2 x perimeter of base x slant height

The base of the pyramid is a square, so its area can be expressed as:

Base area = a^2

The perimeter of the base can be calculated as:

Perimeter of base = 4a

The slant height can be calculated using the Pythagorean theorem:

slant height = sqrt(h^2 + (a/2)^2)

where h is the height of the pyramid.

Substituting these values in the surface area formula, we get:

L = a^2 + 1/2 x 4a x sqrt(h^2 + (a/2)^2)

L = a^2 + 2a x sqrt(h^2 + (a/2)^2)

(b) If a = 10 and b = 12, then the surface area of the pyramid can be calculated as:

L = 10^2 + 2 x 10 x sqrt(h^2 + (10/2)^2)

L = 100 + 20sqrt(h^2 + 25)

Given that L = 192, we can solve for h:

192 - 100 = 20sqrt(h^2 + 25)

92 = 20sqrt(h^2 + 25)

4.6 = sqrt(h^2 + 25)

4.6^2 - 25 = h^2

h^2 = 2.76

h = sqrt(2.76)

h ≈ 1.66

Substituting these values in the surface area formula, we get:

L = 10^2 + 2 x 10 x sqrt(1.66^2 + (10/2)^2)

L ≈ 314.9

Therefore, the surface area of the pyramid is approximately 314.9 square units.

(c) If L = 192 and a = b, then the surface area formula can be simplified as:

L = a^2 + 2a x sqrt(h^2 + (a/2)^2)

192 = a^2 + 2a x sqrt(h^2 + (a/2)^2)

We also know that the height of the pyramid is equal to the side length of the triangular faces. Since the pyramid is a right pyramid, the height and slant height are related by the Pythagorean theorem:

h^2 + (a/2)^

Other Questions
Apply the repeated nearest neighbor algorithm to the graph above. Starting at which vertex or vertices produces the circuit of lowest cost? A ball is drawn randomly from a jar that contains 8 red balls, 7 white balls, and 3 yellow balls. Find the probability of the given event. Write your answers as reduced fractions or whole numbers. (a) P(A red ball is drawn) = (b) P(A white ball is drawn) = (c) P(A yellow ball is drawn) = (d) P(A green ball is drawn) = when approaching a railroad crossing with no warning device, the speed limit is Find the direction and magnitude of : 1. The vector sum A + B [10. 22m, 145. 16]2. The vector A - B, [49. 56m, 157] and 3. The vector difference B - A. [49. 56m, 337]. Element x decays radioactively with a half life of 15 minutes. if there are 960 grams of element x, how long, to the nearest tenth of a minute, would it take the element to decay to 295 grams?y=a(.5)^(t/h) If f(x)4e^x find f(4) rounded to the nearest tenth. Hemoglobin is a protein involved in oxygen transport. Myosin is a protein found in muscles. Which sentence describes these proteins? O A. They can swap their functions when the body requires it. OB. They can change their structures when the body needs more of a different type of protein. O C. They have the same amino acid sequence and are just folded differently. O D. Their structures are based on the amino acid sequences in their polypeptides. Jessica has 300 cm of matenal. She uses 12. 6 cm to make a nght triangular prism She wants to make a second prism that is adilation of the first prism with a scale factor of 3How much more material does Jessica need in order to make the second prism?Select from the drop-down menu to correctly complete the statementcm of material to make the second prismJessica needs an additional Choose please help for this question In the context of the text, how do people create change? what prompted the lovings to seek out additional help to challenge their conviction? cite evidence from this text, your own experience, and other literature, art, or history in your answer. You will always be comfortable throughout the learning process.a. Trueb. False Ronaldo wants to determine if employees would prefer having bagels or donuts at morning meetings. Which method would be best to collect this information? A. Observation B. Report C. Survey D. Database Use the slope of a line formula to find the slope of the following points: (3, 9) and (8, 15) In a nuclear disaster, there are multiple dangerous radioactive isotopes that can be detected. If 91.9% of a particular isotope emitted during a disaster was still present 6 years after the disaster, find the continuous compound rate of decay of this isotope Which practice involves purposeful steps taken by an organization to create employment opportunities for minorities and women? Nondiscrimination O Cultural training Company policy Affirmative action Select the best response to the following questions. What term describes promotion of career and advancement paths for all employees? Cultural training Inclusive audit Company policy Equal opportunity the japanese financial services agency has introduced a stewardship code that calls on investors to exercise their shareholder rights, for example, by pressing for greater returns. in other words, what are investors in japan encouraged to become? 2. Are there any other strategies you think you would struggle with more than others?so, explain why. Complete the paragraph describing different types of self-assessment by filling in the correct words. julio completed a series of self-assessment tests in his career explorations class. his assessment revealed that he has a natural talent for music. his inventory said that he is friendly, and his inventory said that he really likes to be outdoors and around people. 100 Points! Algebra question. Only looking for an answer to B. Please show as much work as possible. Thank you! Photo attached. After an antibiotic is taken, the concentration of the antibiotic in the bloodstream is modeled by the function C(t) = 4te-397, where t is measured in hours and C is measured in ag Use the closed interval methods to mg detremine the maximum concentration of the antibiotic between hours 1 and 7. Write a setence stating your result, round answer to two decimal places, and include units.